Broken stick probability question (variation)

The name of the pictureThe name of the pictureThe name of the pictureClash Royale CLAN TAG#URR8PPP











up vote
0
down vote

favorite












A stick is broken into two at random, then the longer half is broken again into two pieces at random. What is the probability that the three pieces make a triangle?



I've been stumped at this question for a while now, and cannot seem to find a convincing answer on the internet. The answers that I find are usually 0. 386 (which is 2ln2 - 1), but I have worked the answer out to be 0.27865.



I let X be the event of the position of the first cut of a length of unit 1. Then X ~ Unif(0,1).



Then I let Y be the event of the position of the second cut. Finding the PDF of Y is slightly more tricky.



Y is Unif (a/(1-x)) for 0 < x < 0.5 and Unif (a/x) for 0.5 < x < 1



where the normalising constant a, I worked out to be 0.5/ln2.



Now the answers I have seen did not work out this normalising constant, and you will get 0.386 as the probability to create a triangle, but with the normalising constant, I get an answer of 0.27865...



Now, as with these probability questions, you can always make an experiment, and estimate a probability. I was too lazy to do this and found a simulation here:



http://www.sineofthetimes.org/the-broken-stick-puzzle/



I found the simulation to support the 0.38 answer... but then again, I don't know how they created the simulation.



What have I done wrong here? (Or am I right?) Thanks so much!







share|cite|improve this question






















  • Welcome to MSE. For some basic information about writing mathematics at this site see, e.g., basic help on mathjax notation, mathjax tutorial and quick reference, main meta site math tutorial and equation editing how-to.
    – José Carlos Santos
    Aug 9 at 19:17










  • What does "Unif (a/(1-x)) for 0" mean?
    – saulspatz
    Aug 9 at 19:19










  • Some formatting problem with the "less than" sign, sorry.
    – Snow-Covered Island
    Aug 9 at 19:21










  • I think without loss of generality you can consider $X$~$U(frac12, 1)$ and $Y$~$U(0,X)$. Then the question is whether $max(Y,X-Y,1-X)leq frac12$.
    – Arnaud Mortier
    Aug 9 at 19:22







  • 1




    I think you should spell out what you mean by the "normalising constant" $a$. I don't see how and why it should arise. Also, could you link to some of the places where you found the $2ln2-1$ result, and point out what parts of their derivation you don't understand? Then we could avoid replicating the entire derivation here.
    – joriki
    Aug 9 at 19:40














up vote
0
down vote

favorite












A stick is broken into two at random, then the longer half is broken again into two pieces at random. What is the probability that the three pieces make a triangle?



I've been stumped at this question for a while now, and cannot seem to find a convincing answer on the internet. The answers that I find are usually 0. 386 (which is 2ln2 - 1), but I have worked the answer out to be 0.27865.



I let X be the event of the position of the first cut of a length of unit 1. Then X ~ Unif(0,1).



Then I let Y be the event of the position of the second cut. Finding the PDF of Y is slightly more tricky.



Y is Unif (a/(1-x)) for 0 < x < 0.5 and Unif (a/x) for 0.5 < x < 1



where the normalising constant a, I worked out to be 0.5/ln2.



Now the answers I have seen did not work out this normalising constant, and you will get 0.386 as the probability to create a triangle, but with the normalising constant, I get an answer of 0.27865...



Now, as with these probability questions, you can always make an experiment, and estimate a probability. I was too lazy to do this and found a simulation here:



http://www.sineofthetimes.org/the-broken-stick-puzzle/



I found the simulation to support the 0.38 answer... but then again, I don't know how they created the simulation.



What have I done wrong here? (Or am I right?) Thanks so much!







share|cite|improve this question






















  • Welcome to MSE. For some basic information about writing mathematics at this site see, e.g., basic help on mathjax notation, mathjax tutorial and quick reference, main meta site math tutorial and equation editing how-to.
    – José Carlos Santos
    Aug 9 at 19:17










  • What does "Unif (a/(1-x)) for 0" mean?
    – saulspatz
    Aug 9 at 19:19










  • Some formatting problem with the "less than" sign, sorry.
    – Snow-Covered Island
    Aug 9 at 19:21










  • I think without loss of generality you can consider $X$~$U(frac12, 1)$ and $Y$~$U(0,X)$. Then the question is whether $max(Y,X-Y,1-X)leq frac12$.
    – Arnaud Mortier
    Aug 9 at 19:22







  • 1




    I think you should spell out what you mean by the "normalising constant" $a$. I don't see how and why it should arise. Also, could you link to some of the places where you found the $2ln2-1$ result, and point out what parts of their derivation you don't understand? Then we could avoid replicating the entire derivation here.
    – joriki
    Aug 9 at 19:40












up vote
0
down vote

favorite









up vote
0
down vote

favorite











A stick is broken into two at random, then the longer half is broken again into two pieces at random. What is the probability that the three pieces make a triangle?



I've been stumped at this question for a while now, and cannot seem to find a convincing answer on the internet. The answers that I find are usually 0. 386 (which is 2ln2 - 1), but I have worked the answer out to be 0.27865.



I let X be the event of the position of the first cut of a length of unit 1. Then X ~ Unif(0,1).



Then I let Y be the event of the position of the second cut. Finding the PDF of Y is slightly more tricky.



Y is Unif (a/(1-x)) for 0 < x < 0.5 and Unif (a/x) for 0.5 < x < 1



where the normalising constant a, I worked out to be 0.5/ln2.



Now the answers I have seen did not work out this normalising constant, and you will get 0.386 as the probability to create a triangle, but with the normalising constant, I get an answer of 0.27865...



Now, as with these probability questions, you can always make an experiment, and estimate a probability. I was too lazy to do this and found a simulation here:



http://www.sineofthetimes.org/the-broken-stick-puzzle/



I found the simulation to support the 0.38 answer... but then again, I don't know how they created the simulation.



What have I done wrong here? (Or am I right?) Thanks so much!







share|cite|improve this question














A stick is broken into two at random, then the longer half is broken again into two pieces at random. What is the probability that the three pieces make a triangle?



I've been stumped at this question for a while now, and cannot seem to find a convincing answer on the internet. The answers that I find are usually 0. 386 (which is 2ln2 - 1), but I have worked the answer out to be 0.27865.



I let X be the event of the position of the first cut of a length of unit 1. Then X ~ Unif(0,1).



Then I let Y be the event of the position of the second cut. Finding the PDF of Y is slightly more tricky.



Y is Unif (a/(1-x)) for 0 < x < 0.5 and Unif (a/x) for 0.5 < x < 1



where the normalising constant a, I worked out to be 0.5/ln2.



Now the answers I have seen did not work out this normalising constant, and you will get 0.386 as the probability to create a triangle, but with the normalising constant, I get an answer of 0.27865...



Now, as with these probability questions, you can always make an experiment, and estimate a probability. I was too lazy to do this and found a simulation here:



http://www.sineofthetimes.org/the-broken-stick-puzzle/



I found the simulation to support the 0.38 answer... but then again, I don't know how they created the simulation.



What have I done wrong here? (Or am I right?) Thanks so much!









share|cite|improve this question













share|cite|improve this question




share|cite|improve this question








edited Aug 9 at 19:34









joriki

165k10180328




165k10180328










asked Aug 9 at 19:07









Snow-Covered Island

61




61











  • Welcome to MSE. For some basic information about writing mathematics at this site see, e.g., basic help on mathjax notation, mathjax tutorial and quick reference, main meta site math tutorial and equation editing how-to.
    – José Carlos Santos
    Aug 9 at 19:17










  • What does "Unif (a/(1-x)) for 0" mean?
    – saulspatz
    Aug 9 at 19:19










  • Some formatting problem with the "less than" sign, sorry.
    – Snow-Covered Island
    Aug 9 at 19:21










  • I think without loss of generality you can consider $X$~$U(frac12, 1)$ and $Y$~$U(0,X)$. Then the question is whether $max(Y,X-Y,1-X)leq frac12$.
    – Arnaud Mortier
    Aug 9 at 19:22







  • 1




    I think you should spell out what you mean by the "normalising constant" $a$. I don't see how and why it should arise. Also, could you link to some of the places where you found the $2ln2-1$ result, and point out what parts of their derivation you don't understand? Then we could avoid replicating the entire derivation here.
    – joriki
    Aug 9 at 19:40
















  • Welcome to MSE. For some basic information about writing mathematics at this site see, e.g., basic help on mathjax notation, mathjax tutorial and quick reference, main meta site math tutorial and equation editing how-to.
    – José Carlos Santos
    Aug 9 at 19:17










  • What does "Unif (a/(1-x)) for 0" mean?
    – saulspatz
    Aug 9 at 19:19










  • Some formatting problem with the "less than" sign, sorry.
    – Snow-Covered Island
    Aug 9 at 19:21










  • I think without loss of generality you can consider $X$~$U(frac12, 1)$ and $Y$~$U(0,X)$. Then the question is whether $max(Y,X-Y,1-X)leq frac12$.
    – Arnaud Mortier
    Aug 9 at 19:22







  • 1




    I think you should spell out what you mean by the "normalising constant" $a$. I don't see how and why it should arise. Also, could you link to some of the places where you found the $2ln2-1$ result, and point out what parts of their derivation you don't understand? Then we could avoid replicating the entire derivation here.
    – joriki
    Aug 9 at 19:40















Welcome to MSE. For some basic information about writing mathematics at this site see, e.g., basic help on mathjax notation, mathjax tutorial and quick reference, main meta site math tutorial and equation editing how-to.
– José Carlos Santos
Aug 9 at 19:17




Welcome to MSE. For some basic information about writing mathematics at this site see, e.g., basic help on mathjax notation, mathjax tutorial and quick reference, main meta site math tutorial and equation editing how-to.
– José Carlos Santos
Aug 9 at 19:17












What does "Unif (a/(1-x)) for 0" mean?
– saulspatz
Aug 9 at 19:19




What does "Unif (a/(1-x)) for 0" mean?
– saulspatz
Aug 9 at 19:19












Some formatting problem with the "less than" sign, sorry.
– Snow-Covered Island
Aug 9 at 19:21




Some formatting problem with the "less than" sign, sorry.
– Snow-Covered Island
Aug 9 at 19:21












I think without loss of generality you can consider $X$~$U(frac12, 1)$ and $Y$~$U(0,X)$. Then the question is whether $max(Y,X-Y,1-X)leq frac12$.
– Arnaud Mortier
Aug 9 at 19:22





I think without loss of generality you can consider $X$~$U(frac12, 1)$ and $Y$~$U(0,X)$. Then the question is whether $max(Y,X-Y,1-X)leq frac12$.
– Arnaud Mortier
Aug 9 at 19:22





1




1




I think you should spell out what you mean by the "normalising constant" $a$. I don't see how and why it should arise. Also, could you link to some of the places where you found the $2ln2-1$ result, and point out what parts of their derivation you don't understand? Then we could avoid replicating the entire derivation here.
– joriki
Aug 9 at 19:40




I think you should spell out what you mean by the "normalising constant" $a$. I don't see how and why it should arise. Also, could you link to some of the places where you found the $2ln2-1$ result, and point out what parts of their derivation you don't understand? Then we could avoid replicating the entire derivation here.
– joriki
Aug 9 at 19:40










2 Answers
2






active

oldest

votes

















up vote
2
down vote













I think without loss of generality you can consider $X$~$U(frac12, 1)$ and $Y$~$U(0,X)$.



If you set $M=max(Y,X-Y,1-X)$ (the longer side) then the three segments do not form a triangle if and only if $M$ is larger than the sum of the other two sides, that is $M> 1-M$, in other words whether $M> frac12$.



Note that $P(1-X)>frac 12=0$, so $$M>frac 12=max(Y,X-Y)>frac 12$$



Now beginalign*P(M>frac12mid X=x)&=P(Y<x-frac12)+P(frac12<Yleq x)\&=2fracx-frac12x\&=2-frac1xendalign*



So that beginalign*P(M>frac12)&=int_frac12^1P(M>frac12mid X=x)cdot 2dx\&=int_frac12^1 (4-frac2x )dx\&=left[4x-2ln xright]_frac12^1\&=(4-2)+2ln(frac12)\&=2(1-ln 2)endalign*
This is the probability that the three segments do not form a triangle.






share|cite|improve this answer



























    up vote
    0
    down vote













    You do not need a normalising constant



    • If $X=x < frac12$ you have $Y$ conditionally distributed uniformly on $[x,1]$ with density $frac11-x$ and you have a triangle if $Y in left(frac 12, x+frac12right)$ which has conditional probability $fracx1-x$


    • If $X=x > frac12$ you have $Y$ conditionally distributed uniformly on $[0,x]$ with density $frac1x$ and you have a triangle if $Y in left(x-frac12, frac12right)$ which has conditional probability $frac1-xx$


    • So integrating over $x$ uniform on $[0,1]$, you get the overall probability of a triangle being $$int_0^1/2 fracx1-x , dx+ int_1/2^1 frac1-xx , dx = 2log_e(2)-1 approx 0.386$$


    The following simulation in R gives much the same result



    set.seed(1)
    cases <- 10^6
    X <- runif(cases, min=0, max=1)
    Y <- ifelse(X > 1/2, runif(cases, min=0, max=X), runif(cases, min=X, max=1))
    triangle <- ((X > 1/2 & Y < 1/2) | (X < 1/2 & Y > 1/2)) & abs(X-Y) < 1/2
    mean(triangle)
    [1] 0.386273





    share|cite|improve this answer




















      Your Answer




      StackExchange.ifUsing("editor", function ()
      return StackExchange.using("mathjaxEditing", function ()
      StackExchange.MarkdownEditor.creationCallbacks.add(function (editor, postfix)
      StackExchange.mathjaxEditing.prepareWmdForMathJax(editor, postfix, [["$", "$"], ["\\(","\\)"]]);
      );
      );
      , "mathjax-editing");

      StackExchange.ready(function()
      var channelOptions =
      tags: "".split(" "),
      id: "69"
      ;
      initTagRenderer("".split(" "), "".split(" "), channelOptions);

      StackExchange.using("externalEditor", function()
      // Have to fire editor after snippets, if snippets enabled
      if (StackExchange.settings.snippets.snippetsEnabled)
      StackExchange.using("snippets", function()
      createEditor();
      );

      else
      createEditor();

      );

      function createEditor()
      StackExchange.prepareEditor(
      heartbeatType: 'answer',
      convertImagesToLinks: true,
      noModals: false,
      showLowRepImageUploadWarning: true,
      reputationToPostImages: 10,
      bindNavPrevention: true,
      postfix: "",
      noCode: true, onDemand: true,
      discardSelector: ".discard-answer"
      ,immediatelyShowMarkdownHelp:true
      );



      );








       

      draft saved


      draft discarded


















      StackExchange.ready(
      function ()
      StackExchange.openid.initPostLogin('.new-post-login', 'https%3a%2f%2fmath.stackexchange.com%2fquestions%2f2877592%2fbroken-stick-probability-question-variation%23new-answer', 'question_page');

      );

      Post as a guest






























      2 Answers
      2






      active

      oldest

      votes








      2 Answers
      2






      active

      oldest

      votes









      active

      oldest

      votes






      active

      oldest

      votes








      up vote
      2
      down vote













      I think without loss of generality you can consider $X$~$U(frac12, 1)$ and $Y$~$U(0,X)$.



      If you set $M=max(Y,X-Y,1-X)$ (the longer side) then the three segments do not form a triangle if and only if $M$ is larger than the sum of the other two sides, that is $M> 1-M$, in other words whether $M> frac12$.



      Note that $P(1-X)>frac 12=0$, so $$M>frac 12=max(Y,X-Y)>frac 12$$



      Now beginalign*P(M>frac12mid X=x)&=P(Y<x-frac12)+P(frac12<Yleq x)\&=2fracx-frac12x\&=2-frac1xendalign*



      So that beginalign*P(M>frac12)&=int_frac12^1P(M>frac12mid X=x)cdot 2dx\&=int_frac12^1 (4-frac2x )dx\&=left[4x-2ln xright]_frac12^1\&=(4-2)+2ln(frac12)\&=2(1-ln 2)endalign*
      This is the probability that the three segments do not form a triangle.






      share|cite|improve this answer
























        up vote
        2
        down vote













        I think without loss of generality you can consider $X$~$U(frac12, 1)$ and $Y$~$U(0,X)$.



        If you set $M=max(Y,X-Y,1-X)$ (the longer side) then the three segments do not form a triangle if and only if $M$ is larger than the sum of the other two sides, that is $M> 1-M$, in other words whether $M> frac12$.



        Note that $P(1-X)>frac 12=0$, so $$M>frac 12=max(Y,X-Y)>frac 12$$



        Now beginalign*P(M>frac12mid X=x)&=P(Y<x-frac12)+P(frac12<Yleq x)\&=2fracx-frac12x\&=2-frac1xendalign*



        So that beginalign*P(M>frac12)&=int_frac12^1P(M>frac12mid X=x)cdot 2dx\&=int_frac12^1 (4-frac2x )dx\&=left[4x-2ln xright]_frac12^1\&=(4-2)+2ln(frac12)\&=2(1-ln 2)endalign*
        This is the probability that the three segments do not form a triangle.






        share|cite|improve this answer






















          up vote
          2
          down vote










          up vote
          2
          down vote









          I think without loss of generality you can consider $X$~$U(frac12, 1)$ and $Y$~$U(0,X)$.



          If you set $M=max(Y,X-Y,1-X)$ (the longer side) then the three segments do not form a triangle if and only if $M$ is larger than the sum of the other two sides, that is $M> 1-M$, in other words whether $M> frac12$.



          Note that $P(1-X)>frac 12=0$, so $$M>frac 12=max(Y,X-Y)>frac 12$$



          Now beginalign*P(M>frac12mid X=x)&=P(Y<x-frac12)+P(frac12<Yleq x)\&=2fracx-frac12x\&=2-frac1xendalign*



          So that beginalign*P(M>frac12)&=int_frac12^1P(M>frac12mid X=x)cdot 2dx\&=int_frac12^1 (4-frac2x )dx\&=left[4x-2ln xright]_frac12^1\&=(4-2)+2ln(frac12)\&=2(1-ln 2)endalign*
          This is the probability that the three segments do not form a triangle.






          share|cite|improve this answer












          I think without loss of generality you can consider $X$~$U(frac12, 1)$ and $Y$~$U(0,X)$.



          If you set $M=max(Y,X-Y,1-X)$ (the longer side) then the three segments do not form a triangle if and only if $M$ is larger than the sum of the other two sides, that is $M> 1-M$, in other words whether $M> frac12$.



          Note that $P(1-X)>frac 12=0$, so $$M>frac 12=max(Y,X-Y)>frac 12$$



          Now beginalign*P(M>frac12mid X=x)&=P(Y<x-frac12)+P(frac12<Yleq x)\&=2fracx-frac12x\&=2-frac1xendalign*



          So that beginalign*P(M>frac12)&=int_frac12^1P(M>frac12mid X=x)cdot 2dx\&=int_frac12^1 (4-frac2x )dx\&=left[4x-2ln xright]_frac12^1\&=(4-2)+2ln(frac12)\&=2(1-ln 2)endalign*
          This is the probability that the three segments do not form a triangle.







          share|cite|improve this answer












          share|cite|improve this answer



          share|cite|improve this answer










          answered Aug 9 at 19:57









          Arnaud Mortier

          19.2k22159




          19.2k22159




















              up vote
              0
              down vote













              You do not need a normalising constant



              • If $X=x < frac12$ you have $Y$ conditionally distributed uniformly on $[x,1]$ with density $frac11-x$ and you have a triangle if $Y in left(frac 12, x+frac12right)$ which has conditional probability $fracx1-x$


              • If $X=x > frac12$ you have $Y$ conditionally distributed uniformly on $[0,x]$ with density $frac1x$ and you have a triangle if $Y in left(x-frac12, frac12right)$ which has conditional probability $frac1-xx$


              • So integrating over $x$ uniform on $[0,1]$, you get the overall probability of a triangle being $$int_0^1/2 fracx1-x , dx+ int_1/2^1 frac1-xx , dx = 2log_e(2)-1 approx 0.386$$


              The following simulation in R gives much the same result



              set.seed(1)
              cases <- 10^6
              X <- runif(cases, min=0, max=1)
              Y <- ifelse(X > 1/2, runif(cases, min=0, max=X), runif(cases, min=X, max=1))
              triangle <- ((X > 1/2 & Y < 1/2) | (X < 1/2 & Y > 1/2)) & abs(X-Y) < 1/2
              mean(triangle)
              [1] 0.386273





              share|cite|improve this answer
























                up vote
                0
                down vote













                You do not need a normalising constant



                • If $X=x < frac12$ you have $Y$ conditionally distributed uniformly on $[x,1]$ with density $frac11-x$ and you have a triangle if $Y in left(frac 12, x+frac12right)$ which has conditional probability $fracx1-x$


                • If $X=x > frac12$ you have $Y$ conditionally distributed uniformly on $[0,x]$ with density $frac1x$ and you have a triangle if $Y in left(x-frac12, frac12right)$ which has conditional probability $frac1-xx$


                • So integrating over $x$ uniform on $[0,1]$, you get the overall probability of a triangle being $$int_0^1/2 fracx1-x , dx+ int_1/2^1 frac1-xx , dx = 2log_e(2)-1 approx 0.386$$


                The following simulation in R gives much the same result



                set.seed(1)
                cases <- 10^6
                X <- runif(cases, min=0, max=1)
                Y <- ifelse(X > 1/2, runif(cases, min=0, max=X), runif(cases, min=X, max=1))
                triangle <- ((X > 1/2 & Y < 1/2) | (X < 1/2 & Y > 1/2)) & abs(X-Y) < 1/2
                mean(triangle)
                [1] 0.386273





                share|cite|improve this answer






















                  up vote
                  0
                  down vote










                  up vote
                  0
                  down vote









                  You do not need a normalising constant



                  • If $X=x < frac12$ you have $Y$ conditionally distributed uniformly on $[x,1]$ with density $frac11-x$ and you have a triangle if $Y in left(frac 12, x+frac12right)$ which has conditional probability $fracx1-x$


                  • If $X=x > frac12$ you have $Y$ conditionally distributed uniformly on $[0,x]$ with density $frac1x$ and you have a triangle if $Y in left(x-frac12, frac12right)$ which has conditional probability $frac1-xx$


                  • So integrating over $x$ uniform on $[0,1]$, you get the overall probability of a triangle being $$int_0^1/2 fracx1-x , dx+ int_1/2^1 frac1-xx , dx = 2log_e(2)-1 approx 0.386$$


                  The following simulation in R gives much the same result



                  set.seed(1)
                  cases <- 10^6
                  X <- runif(cases, min=0, max=1)
                  Y <- ifelse(X > 1/2, runif(cases, min=0, max=X), runif(cases, min=X, max=1))
                  triangle <- ((X > 1/2 & Y < 1/2) | (X < 1/2 & Y > 1/2)) & abs(X-Y) < 1/2
                  mean(triangle)
                  [1] 0.386273





                  share|cite|improve this answer












                  You do not need a normalising constant



                  • If $X=x < frac12$ you have $Y$ conditionally distributed uniformly on $[x,1]$ with density $frac11-x$ and you have a triangle if $Y in left(frac 12, x+frac12right)$ which has conditional probability $fracx1-x$


                  • If $X=x > frac12$ you have $Y$ conditionally distributed uniformly on $[0,x]$ with density $frac1x$ and you have a triangle if $Y in left(x-frac12, frac12right)$ which has conditional probability $frac1-xx$


                  • So integrating over $x$ uniform on $[0,1]$, you get the overall probability of a triangle being $$int_0^1/2 fracx1-x , dx+ int_1/2^1 frac1-xx , dx = 2log_e(2)-1 approx 0.386$$


                  The following simulation in R gives much the same result



                  set.seed(1)
                  cases <- 10^6
                  X <- runif(cases, min=0, max=1)
                  Y <- ifelse(X > 1/2, runif(cases, min=0, max=X), runif(cases, min=X, max=1))
                  triangle <- ((X > 1/2 & Y < 1/2) | (X < 1/2 & Y > 1/2)) & abs(X-Y) < 1/2
                  mean(triangle)
                  [1] 0.386273






                  share|cite|improve this answer












                  share|cite|improve this answer



                  share|cite|improve this answer










                  answered Aug 9 at 23:04









                  Henry

                  93.3k470147




                  93.3k470147






















                       

                      draft saved


                      draft discarded


























                       


                      draft saved


                      draft discarded














                      StackExchange.ready(
                      function ()
                      StackExchange.openid.initPostLogin('.new-post-login', 'https%3a%2f%2fmath.stackexchange.com%2fquestions%2f2877592%2fbroken-stick-probability-question-variation%23new-answer', 'question_page');

                      );

                      Post as a guest













































































                      這個網誌中的熱門文章

                      Is there any way to eliminate the singular point to solve this integral by hand or by approximations?

                      Why am i infinitely getting the same tweet with the Twitter Search API?

                      Carbon dioxide